Chapter 1: Fuel Metabolism and Nutrition - Basic Principles

अब Quizwiz के साथ अपने होमवर्क और परीक्षाओं को एस करें!

Muscle glycogen cannot contribute to blood glucose levels because it *lacks the enzyme glucose 6-phosphatase*, which converts glucose 6-phosphate to glucose that can be released into the blood. Therefore, glucose cannot be released from muscle cells.

BONUS QUESTION: Why can't muscle glycogen contribute to blood glucose levels?

The liver lacks a critical enzyme known as acetoacetate succinyl-CoA transferase, which is used during ketone body breakdown.

BONUS QUESTION: Why can't the liver utilize ketone bodies for energy?

*The answer is A.* *By 24 hours after a meal, hepatic (liver) gluconeogenesis is the major source of blood glucose* because hepatic glycogen stores have been nearly depleted. Muscle and other tissues lack an enzyme necessary to convert glycogen or amino acids to glucose (thus, B is incorrect). The liver is the only significant source of blood glucose. Glucose is synthesized in the liver from amino acids (provided by protein degradation), from glycerol (provided by hydrolysis of triacylglycerols in adipose tissue), and from lactate (provided by anaerobic glycolysis in red blood cells and other tissues). Glucose cannot be synthesized from fatty acids or ketone bodies (thus, D and E are incorrect).

By 24 hours after a meal, which one of the following is most likely to occur? (A) Gluconeogenesis in the liver will be the major source of blood glucose. (B) Muscle glycogenolysis provides glucose to the blood. (C) Muscles convert amino acids to blood glucose. (D) Fatty acids released from adipose tissue provide carbon for synthesis of glucose. (E) Ketone bodies provide carbon for gluconeogenesis.

*The answer is B.* Both proteins and carbohydrates contain 4 cal/g, so to make up approximately 1,200 cal/day the total intake of proteins and carbohydrates would have to be 300 g/day. Since the runner wants to split the calories equally between protein and carbohydrates, 150 g of each is a better answer than splitting the 300 g into 100 g of one nutrient, and 200 g of another nutrient

A 15-year-old girl has gone to the nutritionist as she is concerned about losing weight. She is 5′7′′ tall and weighs 128 lb, down from 135 lb 3 weeks ago. She explains that she had made the cross-country team at her high school, and over the past 3 weeks her running has increased from about 1.5 miles/day to 10 miles/day. After learning that her diet was deficient in calories, the runner decided to make up the deficit by eating equal amounts (in term of calories) of carbohdyrates and proteins, but no fat or alcohol. How many grams of carbs and proteins would she have to add to her diet in order to stop losing weight? (A) 100 g of each (B) 150 g of each (C) 200 g of each (D) 100 g of carbohydrates, 200 g of protein (E) 100 g of protein, 200 g of carbohydrate

*The answer is C.* Each pound of weight is equivalent to about 3,500 cal. The runner had lost 7 lb, for a total deficit of 24,500 cal. As she lost that weight over 21 days, her daily deficit, if evenly distributed over the course of the three weeks, was 1,167 cal/day.

A 15-year-old girl has gone to the nutritionist as she is concerned about losing weight. She is 5′7′′ tall and weighs 128 lb, down from 135 lb 3 weeks ago. She explains that she had made the cross-country team at her high school, and over the past 3 weeks her running has increased from about 1.5 miles/day to 10 miles/day. Assuming that the patient's weight loss over the past 3 weeks was equally distributed over that time period, how many calories per day was she deficient in her diet? (A) 800 (B) 1,000 (C) 1,200 (D) 1,400 (E) 1,600

*The answer is B.* Blood glucose decreases because insulin stimulates the transport of glucose into muscle and adipose cells and stimulates the conversion of glucose to glycogen and triacylglycerols in the liver. Ketone bodies are not made from blood glucose. During fasting, when the liver is producing ketone bodies, it is also synthesizing glucose. Carbon for ketone body synthesis comes from fatty acids. Insulin stimulates glycogen synthesis, not glycogenolysis. Muscle glycogen is not converted to blood glucose.

A 16-year-old patient with Type 1 diabetes mellitus was admitted to the hospital with a blood glucose level of 400 mg/dL. (The reference range for blood glucose is 80 to 100 mg/dL.) One hour after an insulin infusion was begun, her blood glucose level had decreased to 320 mg/dL. One hour later, it was 230 mg/dL. The patient's glucose level decreased because the infusion of insulin led to which one of the following? (A) The stimulation of the transport of glucose across the cell membranes of the liver and brain (B) The stimulation of the conversion of glucose to glycogen and triacylglycerol in the liver (C) The inhibition of the synthesis of ketone bodies from blood glucose (D) The stimulation of glycogenolysis in the liver (E) The inhibition of the conversion of muscle glycogen to blood glucose

*The answer is C.* In an attempt to save muscle tissue (amino acids used for gluconeogenesis), the brain in starvation mode will utilize ketone bodies for a portion of its energy needs. Liver glycogen stores would be depleted under the conditions described. Heme is not used for energy production, and produces bilirubin when degraded, which cannot be used to generate energy or ketone bodies. Muscle glycogen cannot contribute to blood glucose levels, as muscle tissue lacks the enzyme that allows free glucose to be produced within the muscle.

A 27-year-old male got lost while hiking in Yosemite National Park. He was found 8 days later. He had nothing to eat and only water to drink before being rescued. The man's brain would attempt to decrease consumption of glucose and increase consumption of ketones in order to protect the breakdown (catabolism) of which one of the following? (A) Muscle glycogen (B) Liver glycogen (C) Muscle protein (D) Red blood cells (to provide heme) (E) Adipose triacylglycerol

*The answer is B.* *Red blood cells lack mitochondria, so they can use only glucose for fuel (fatty acids and ketone bodies require mitochondrial proteins for their oxidative pathways).* The brain can also use ketone bodies, along with glucose. The liver can use glucose, fatty acids, and amino acids as energy sources. The heart can use glucose, fatty acids, amino acids, and lactic acid as potential energy sources.

A 27-year-old male got lost while hiking in Yosemite National Park. He was found 8 days later. He had nothing to eat and only water to drink before being rescued. Which cell can only use glucose for energy needs? (A) Brain (B) Red blood cells (C) Hepatocyte (D) Heart (E) Muscle

*The answer is D.* Eicosapentaenoic acid (EPA, a 20-carbon fatty acid containing five double bonds) can be derived from an essential fatty acid found in fish oils (linolenic acid), and is a precursor of eicosanoids (prostaglandins, leukotrienes, and thromboxanes). EPA is also ingested from fish oils. Lactic acid is produced from muscle and red blood cells, and is not an essential nutrient. Palmitic acid (a fatty acid containing 16 carbons, with no double bonds), oleic acid (a fatty acid containing 18 carbons, with one double bond), and stearic acid (a fatty acid containing 18 carbons, with no double bonds) can all be synthesized by the mammalian liver through the normal pathway of fatty acid synthesis.

A 27-year-old male got lost while hiking in Yosemite National Park. He was found 8 days later. He had nothing to eat and only water to drink before being rescued. Which one of the following is an essential nutrient that he has not received over the last 8 days? (A) Lactic acid (B) Oleic acid (C) Steric acid (D) EPA (E) Palmitic acid

*The answer is A.* Albumin, though nonspecific, is considered the standard for assessing overall protein malnutrition. Albumin is made by the liver and is found in the blood. It acts as a nonspecific carrier of fatty acids and other hydrophobic molecules. When amino acid levels become limiting, the liver reduces its levels of protein synthesis, and a reduction in albumin levels in the circulation is an indication of liver dysfunction. Ferritin is an iron storage protein within tissues, and its circulating levels are low at all times. Creatinine is a degradation product of creatine phosphate (an energy storage molecule in muscle), and its presence in the circulation reflects the rate of creatinine clearance by the kidney. High levels of creatinine indicate a renal insufficiency. Creatine phosphokinase is a muscle enzyme that is released into circulation only when there is damage to the muscle. Blood urea nitrogen indicates the rate of amino acid metabolism to generate urea, but does not indicate protein malnutrition.

A 27-year-old male got lost while hiking in Yosemite National Park. He was found 8 days later. He had nothing to eat and only water to drink before being rescued. Which one of the following lab tests should be run on the patient to determine whether he is suffering from overall protein malnutrition? (A) Albumin (B) Blood urea nitrogen (BUN) (C) Creatinine (D) Ferritin (E) Creatine phosphokinase (CPK)

*The answer is D.* The glycerol component of triacylglycerol would be the major contributor of carbons for gluconeogenesis among the answer choices provided. Substrates for hepatic gluconeogenesis are lactate (from red blood cells), amino acids (from muscle), and glycerol (from adipose tissue). Fatty acids would be used for energy, but the carbons of fatty acids cannot be used for the net synthesis of glucose. Hepatic glycogen stores are exhausted about 30 hours after the initiation of the fast, and muscle glycogen stores contribute only to muscle energy needs and not to the maintenance of blood glucose levels.

A 27-year-old male got lost while hiking in Yosemite National Park. He was found 8 days later. He had nothing to eat and only water to drink before being rescued. Which one of the following would be his primary source of carbons for maintaining blood glucose levels when he was found? (A) Liver glycogen (B) Muscle glycogen (C) Fatty acids (D) Triacylglycerol (E) Ketone bodies

*The answer is D.* This person is experiencing ketotic hypoglycemia. In starvation conditions, protein catabolism in muscle provides gluconeogenic amino acids. Therefore, if the protein is unable to break down proteins, gluconeogenesis will decrease, leading to hypoglycemia. When alanine is given, blood glucose will increase because alanine will be used to gluconeogenesis.

A 3-year-old girl who is small for her age has had multiple hypoglycemic episodes associated with moderate fasting during periods of illness. In her most recent episode associated with influenza, her parents slept late, and at 10 am were unable to rouse the child from sleep. Blood and urine samples collected at the E.R. reveal marked hypoglycemia, ketonuria, and ketonemia along with an appropriately low insulin level. The blood alanine level is abnormally low; however, infusion of alanine produces a rapid rise in blood glucose. The defect most likely responsible for these symptoms is found in which of the following pathways? A. Fatty acid oxidation B. Gluconeogenesis C. GIycogenoIysis D. Protein catabolism in muscle E. Triglyceride breakdown in adipose tissue

*The answer is B.* Beta hydroxybutyrate is a ketone body. Ketone bodies serve as alternative fuel for the brain during prolonged fasting or starvation. Fatty acids, due to long hydrophobic chains, cannot cross the blood-brain barrier. Glycerol is a substrate of gluconeogenesis. In fact, during prolonged fasting, this is the only substrate left to provide glucose through pathway of gluconeogenesis. It can also be oxidized through glycolysis upon phosphorylation. Beta carotene is a provitamin; it is not a source of energy. Alanine is a transporter of amino groups of amino acids from the muscle (glucose-alanine cycle), but it cannot be used as an alternative source of energy.

A 30-year-old man has been fasting for religious reasons for several days. His brain has reduced its need for glucose by using which of the following substances as an alternate source of energy? (A) Fatty acids (B) Beta-hydroxybutyrate (C) Glycerol (D) Beta-carotene

*The answer is D.* The lack of one essential amino acid will lead to a negative nitrogen balance due to increased protein degradation to supply that amino acid for the ongoing protein synthesis. Of the amino acids listed, only threonine is an essential amino acid (alanine can be synthesized from pyruvate [which can be derived from glucose], arginine is produced in the urea cycle using aspartic acid and the amino acid ornithine, glycine is derived from serine, and serine is derived from 3 phosphoglycerate, which can be produced from glucose).

A 32-year-old male is on a weight-maintenance diet, so he does not want to lose or gain any weight. Which amino acid must be present in the diet so the patient does not go into a negative nitrogen balance? (A) Alanine (B) Arginine (C) Glycine (D) Threonine (E) Serine

*The answer is C.* After several hours of fasting, the body's glycogen stores are depleted and gluconeogenesis Is required to maintain plasma glucose levels. With prolonged starvation, the body limits its reliance on gluconeogenesis in an effort to conserve protein, and resorts instead to hepatic ketone body synthesis. The brain, kidneys, cardiac muscle, end skeletal muscle (Choices A, B, D & E) can all utilize glucose and/or ketones for energy. (Initially, the heart and skeletal muscle consume primarily ketone bodies to preserve glucose for the brain. After prolonged starvation, however, even the brain utilizes ketone bodies for energy. During starvation, the continuous supply of energy that ketone bodies provide is especially important to brain functioning, because the brain has no glycogen or triglyceride stores. Ketone bodies yield energy when they are converted to acetyl CoA. Erythrocytes cannot use ketone bodies for energy because they lack mitochondria. The liver is also unable to utilize ketone bodies for energy because it lacks the enzyme succinyl CoA-acetoacetate CoA transferee (thiophorase). which is required to convert acetoacetate to acetoacetyl CoA. *Educational Objective:* Erythrocytes and other cells lacking mitochondria cannot utilize ketone bodies for energy. While hepatocytes contain mitochondria, they also cannot utilize ketone bodies because they lack the enzyme succinyl CoA acetoacelate CoA transferase (thiophorase).

A 34-year-old mountain climber is stranded for one week without food or supplies before being rescued. On medical evaluation, he has a high plasma concentration of ketone bodies. Which of the following cannot use ketone bodies for energy? A. Skeletal muscle B. Heart muscle C. Erythrocytes D. Brain tissue E. Renal tissue

*The answer is B.* The recommended total fat intake is less than 30% of total calories. His total caloric consumption was 4,110 kcal/day (carbohydrate, 4 x 585 = 2,340 kcal; protein, 150 x 4 = 600 kcal; fat, 95 x 9 = 855 kcal; alcohol, 45 x 7 = 315 kcal) (thus, A is incorrect). His fat intake was 21% (855 ÷ 4,110) of his total caloric intake. His alcohol intake was 7.7% (315 ÷ 4,110) (thus, C is incorrect). His protein intake was well above the RDA of 0.8 g/kg body weight (thus, D is incorrect). His RMR is roughly 24 kcal/day/kg body weight or 2,880 kcal/day (it will actually be less because he is obese and has a greater proportion of metabolically less active tissue than the average 70-kg man). His daily energy expenditure is about 3,744 kcal/day (1.3 x 2,880) or less. Thus, his intake is greater than his expenditure, and he is in positive caloric balance and is gaining weight (thus, E is incorrect).

A 35-year-old sedentary male patient weighing 120 kg was experiencing angina (chest pain) and other signs of coronary artery disease. His physician, in consultation with a registered dietician, conducted a 3-day dietary recall. The patient consumed an average of 585 g of carbohydrate, 150 g of protein, and 95 g of fat each day. In addition, he drank 45 g of alcohol. The patient's diet is best described by which one of the following? A. He consumed between 2,500 and 3,000 kcal/day. B. He had a fat intake within the range recommended in current dietary guidelines (i.e., the year 2010). C. He consumed 50% of his calories as alcohol. D. He was deficient in protein intake. E. He was in negative caloric balance.

*The answer is D.* Dietary energy comes predominantly from protein, carbohydrate, end fat. Metabolism yields 4 Calories (Cal) per gram of protein or carbohydrate and 9 Cal per gram of fat. Ethanol yields 7 Cal per gram. This patient is instructed to consume 3000 Cal per day, 900 (30%) of which are to be from protein. Because 1 g of protein yields 4 Cal of energy, this patient should consume (900 Cal /4 Cd) = 225 g/day of protein. *Educational Objective:* Metabolism of 1 g of protein or carbohydrate produces 4 Calories of energy. Metabolism of 1 g of fat produces 9 Calories.

A 46-year-old obese man is referred to a dietitian for evaluation of his food intake. He has been trying to lose weight but has been unsuccessful. The patient is 172.7 cm (5 ft 8 in) tall and weighs 113 kg (250 lb). Analysis of his food intake shows that he is consuming 3800 Calories a day. The diet ran recommends increasing physical activity and implementing a dietary plan. In the first phase, the patient is advised to reduce his daily dietary intake to 3,000 Calories, with 30% coming from protein. How much protein per day will this patient consume on the new dietary plan? (A) 130 g (B) 160 g (C) 180 g (D) 225 g (E) 250 g

*The answer is B.* The patient has become hypoglycemic due to excessive release of insulin from the pancreas. Glipizide (glucotrol) is a sulfonylurea drug that stimulates insulin release from the pancreas. If taken in excess, the insulin will promote fat and muscle cells to take up glucose from the circulation, leading to hypoglycemia and insufficient blood glucose levels for normal brain function. Lactic acidosis may result from such an overdose, but it would be secondary to the hypoglycemic symptoms observed. Elevated ammonia levels would not occur, as glipizide does not alter amino acid metabolism. The high levels of insulin released by the drug would inhibit fatty acid release from the adipocytes, and therefore the precursors for ketone body synthesis are not available, and ketoacidosis would not occur.

A 50-year-old male with Type 2 diabetes is taking glipizide to help control his blood sugar levels. On one day he could not remember if he had taken the medication, so he accidently took a second dose of the drug. Two hours later, he suddenly develops irritability, tremors, tachycardia, and lightheadedness. The patient is experiencing which one of the following due to his drug overdose? (A) Hyperglycemia (B) Hypoglycemia (C) Lactic acidosis (D) Ketoacidosis (E) Hyperammonemia

*The answer is B*. The patient's weight, age, and activity put him at *higher risk for insulin resistance syndrome*. The entire syndrome includes hypertension, diabetes mellitus (type 2), decreased high-density lipoprotein levels, increased triglyceride levels, increased urate, increased levels of plasminogen activator inhibitor 1, nonalcoholic fatty liver, central obesity, and polycystic ovary syndrome (PCOS) (in females). Insulin resistance syndrome leads to early atherosclerosis throughout the entire body. The patient is not at increased risk for diabetes mellitus, type 1, as that is the result of an autoimmune condition that destroys the β cells of the pancreas such that insulin can no longer be produced. The lifestyle exhibited by the patient has not been linked to autoimmune disorders. Gaucher disease is a disorder of the enzyme β-glucocerebrosidase, and is an autosomal recessive disorder. Since this disease is an inherited disorder, the patient's lifestyle does not increase his risk of having this disease. The patient's increasing weight might lead to increased blood pressure, but not to reduced blood pressure. Sickle cell disease is another autosomal recessive disorder leading to an altered β-globin gene product, and like Gaucher disease, it is an inherited disorder that is not altered by the patient's lifestyle.

A 50-year-old male with a "pot belly" and a strong family history of heart attacks is going to his physician for advice on how to lose weight. He weighs 220 lb (100 kg) and is about 6' tall (1.85 m). His lifestyle can be best described as sedentary. For which of the following disease processes is this patient at higher risk? (A) Diabetes mellitus, type 1 (B) Insulin resistance syndrome (C) Gaucher disease (D) Low blood pressure (E) Sickle cell disease

*The answer is C.* The DEE is equal to the BMR plus physical activity factor. For the patient in question, the BMR = 24 kcal/kg/day x 100 kg, or 2,400 kcal/day. Since the patient is sedentary, the activity level is 30% that of the BMR, or 720 kcal/day. The overall daily needs are therefore 2,400 + 720 kcal/day, or 3,120 kcal/day. If the patient consumes <3,000 kcal/day, or increases his physical activity level, then weight loss would result.

A 50-year-old male with a "pot belly" and a strong family history of heart attacks is going to his physician for advice on how to lose weight. He weighs 220 lb (100 kg) and is about 6' tall (1.85 m). His lifestyle can be best described as sedentary. How many kilocalories per day would the patient need to maintain this weight? (A) 2,400 (B) 2,620 (C) 3,120 (D) 3,620 (E) 3,950

*The answer is C.* The patient is in the *overweight (preobese*) category with a BMI of 29. As indicated, a BMI of <18.5 is the underweight category, a BMI between 18.5 and 24.9 is the healthy range, a BMI between 25 and 30 is the overweight (preobese) category, and any BMI of 30 or above is considered the obese range. Class I obese is between 30 and 35, whereas class II obese is between 35 and 40. Class III obesity, or morbidly obese, is the classification for individuals with a BMI of 40 or higher.

A 50-year-old male with a "pot belly" and a strong family history of heart attacks is going to his physician for advice on how to lose weight. He weighs 220 lb (100 kg) and is about 6' tall (1.85 m). His lifestyle can be best described as sedentary. Into which of the following categories does his BMI place him? (A) Underweight (B) Healthy (C) Overweight (preobese) (D) Obese (class I) (E) Obese (class II)

*The answer is B.* The BMI is equal to kg/m², which in this case is equal to 100/1.85², which is about 29.

A 50-year-old male with a "pot belly" and a strong family history of heart attacks is going to his physician for advice on how to lose weight. He weighs 220 lb (100 kg) and is about 6' tall (1.85 m). His lifestyle can be best described as sedentary. What is this patient's BMI? (A) 24 (B) 29 (C) 31 (D) 36 (E) 40

*The answer is A.* Decreased serum albumin could have several causes, including hepatic disease that decreases the ability of the liver to synthesize serum proteins, protein malnutrition, marasmus, or diseases that affect the ability of the intestine to digest protein and absorb the amino acids. However, his BMI is in the healthy weight range (thus, B and D are incorrect). His normal CHI indicates that he has not lost muscle mass and is, therefore, not suffering from protein malnutrition (thus, B, C, D, and E are incorrect).

A hospitalized patient had low levels of serum albumin and high levels of blood ammonia. His CHI was 98%. His BMI was 20.5. BUN was not elevated, consistent with normal kidney function. The diagnosis most consistent with these finding is which one of the following? A. A loss of hepatic function (e.g., alcohol-induced cirrhosis) B. Anorexia nervosa C. Kwashiorkor (protein malnutrition) D. Marasmus (protein-calorie malnutrition) E. Decreased absorption of amino acids by intestinal epithelial cells (e.g., celiac disease)

*The answer is A.* The patient's BMI is in the obese range, with large abdominal fat deposits. He needs to decrease his intake of total calories because an excess of calories ingested as carbohydrate, fat, or protein results in deposition of triacylglycerols in adipose tissue. If he keeps his total caloric intake the same, substitution of one type of food for another will help very little with weight loss. (However, a decreased intake of fat may be advisable for other reasons). Limited food diets, such as the ice cream and sherry diet, or a high protein diet of shrimp, work if they decrease appetite and, therefore, ingestion of total calories.

A male patient exhibited a BMI of 33 kg/m² and a waist circumference of 47 in. What dietary therapy would you consider most helpful? A. Decreased intake of total calories, because all fuels can be converted to adipose tissue triacylglycerols. B. The same amount of total calories, but substitution of carbohydrate calories for fat calories. C. The same amount of total calories, but substitution of protein calories for fat calories. D. A pure-fat diet, because only fatty acids synthesized by the liver can be deposited as adipose triacylglycerols. E. A limited food diet, such as the ice cream and sherry diet.

*The answer is A.* After a meal of carbohydrates (the major ingredient of pretzels), *glycogen is stored in the liver and in muscle, and triacylglycerols are stored in adipose tissue*. Owing to the rise in glucose level in the blood (from the carbohydrates in the pretzels), *insulin is released from the pancreas* and the level of glucagon in the blood decreases. Since blood glucose levels have increased, there is no longer a need for the liver to synthesize glucose, and gluconeogenesis decreases. The change in insulin-to-glucagon ratio also inhibits the breakdown of triacylglycerols and favors their synthesis. The brain oxidizes glucose to CO₂ and H₂O, whereas the red blood cells produce lactate from glucose, since red blood cells cannot carry out aerobic metabolism.

A medical student has been studying for exams, and neglects to eat anything for 12 hours. At this point, the student opens a large bag of pretzels and eats every one of them in a short period. Which one of the following effects will this meal have on the student's metabolic state? (A) Liver glycogen stores will be replenished. (B) The rate of gluconeogenesis will be increased. (C) The rate at which fatty acids are converted to adipose triacylglycerols will be reduced. (D) Blood glucagon levels will increase. (E) Glucose will be oxidized to lactate by the brain and to CO₂ and H₂O by the red blood cells.

*The answer is D.* In the starvation state, *muscle decreases the use of ketone bodies*, causing an *elevation of ketone bodies in the bloodstream*. The *brain uses the ketone bodies for energy and uses less glucose*, which *decreases the need for gluconeogenesis*, thus sparing muscle protein degradation to provide the precursors for gluconeogenesis. *Red blood cells cannot use ketone bodies and must utilize glucose.* Therefore, the use of glucose by red blood cells would be unchanged under these conditions.

A patient is brought to the emergency room after being found by search and rescue teams. He was mountain climbing, got caught in a sudden snowstorm, and had to survive in a cave. He had no food for 6 days. In adapting to these conditions, which metabolic process has increased rather than decreased? (A) The brain's use of glucose (B) Muscle's use of ketone bodies (C) The red blood cells' use of glucose (D) The brain's use of ketone bodies (E) The red blood cells' use of ketone bodies (F) Muscle's use of glucose

*The answer is D.* Once insulin is injected, glucose transport into the peripheral tissues will be enhanced. If the patient does not eat, *the normal fasting level of glucose will drop even further because of the injection of insulin, which increases the movement of glucose into muscle and fat cells*. The patient becomes *hypoglycemic*8, as a result of which epinephrine is released from the adrenal medulla. This, in turn, leads to the signs and symptoms associated with high levels of epinephrine in the blood. Answers A and B are incorrect because as glucose levels drop, glucagon will be released from the pancreas to raise blood glucose levels, which would alleviate the symptoms. Answer E is incorrect because ketone body production does not produce hypoglycemic symptoms, nor would they be significantly elevated only a few hours after the insulin shock the patient is experiencing.

A patient with type 1 diabetes mellitus takes an insulin injection before eating dinner but then gets distracted and does not eat. Approximately 3 hours later, the patient becomes shaky, sweaty, and confused. These symptoms have occurred because of which of the following? (A) Increased glucagon release from the pancreas (B) Decreased glucagon release from the pancreas (C) High blood glucose levels (D) Low blood glucose levels (E) Elevated ketone body levels

*The answer is A.* The liver will produce ketone bodies when fatty acid oxidation is increased, which occurs when glucagon is the predominant hormone (glucagon leads to fatty acid release from the fat cells, for oxidation in the liver and muscle). This would be the case in an individual who cannot produce insulin and is not taking insulin injections. However, in this situation, the ketone bodies are not being used by the nervous system (brain) because of the high levels of glucose in the circulation. This leads to severely elevated ketone levels due to nonuse. The glucose is high because, in the absence of insulin, muscle and fat cells are not using the glucose in circulation as an energy source. Recall, although the liver produces ketone bodies, it lacks a necessary enzyme to use ketone bodies as an energy source. There is no relation between blood urea nitrogen levels and the rate of ketone body production. The muscle reduces its use of ketone bodies under these conditions, but not of fatty acids.

A physician is treating a type 1 diabetic patient who has neglected to take his or her insulin for 5 days. The patient demonstrates elevated blood glucose and ketone body levels. Ketone bodies are elevated due to which one of the following? (A) Elevated glucose levels (B) Reduced BUN (C) Decreased fatty acid release from the adipocyte (D) Inhibition of liver oxidation of ketone bodies (E) Reduced muscle utilization of fatty acids

*The answer is C.* The children are exhibiting the effects of *kwashiorkor*, a disorder resulting from *adequate calorie intake but insufficient calories from protein*. This results in the liver producing less serum albumin (due to the lack of essential amino acids), which affects the osmotic balance of the blood and the fluid in the interstitial spaces. Owing to the reduction in osmotic pressure of the blood, water leaves the blood and enters the interstitial spaces, producing *edema* in the children (which leads to the expanded abdomen). The children are degrading muscle protein to allow the synthesis of new protein (due to a lack of essential amino acids), and this leads to the wasting of the arms and legs of children with this disorder. The children will exhibit normal or slightly elevated levels of ketone bodies and fatty acids in the blood, as the diet is calorie sufficient. Glycogen levels may only be slightly reduced (since the diet is calorie sufficient), but glycogen is not found in the blood. Glucose levels will be only slightly reduced, as gluconeogenesis will keep glucose levels near normal.

A physician working in a refugee camp in Africa notices a fair number of children with emaciated arms and legs, yet a large protruding stomach and abdomen. An analysis of the children's blood would show significantly reduced levels of which one of the following as compared with those in a healthy child? (A) Glucose (B) Ketone bodies (C) Albumin (D) Fatty acids (E) Glycogen

*The answer is B.* In order to answer this question, one first needs to calculate the basic daily energy needs of the student. At 175 lb (79.55 kg) one can estimate his BMR as 1,910 cal/day (79.55 kg multiplied by 24 cal/day/kg). Being sedentary, the activity factor is 1.3, for a total daily caloric need of 2,480 cal/day. Current nutritional guidelines indicate that no more than 30% of one's daily calories should be fat, so the maximum caloric intake for fat should be 750 cal, which is about 80 g of fat (fat contains 9 cal/g). These data alone eliminate answers A and C. Answer E has insufficient total calories (450 from fat, 100 from protein, and 900 from carbohydrates, for a total of 1,450) for the needs of the student, and can also be eliminated. Answer D contains too much ethanol (420 cal out of a total of 2,780) and too many calories. Thus, answer B is correct, in which the 80 g of fat provide 720 cal, the 125 g of protein provides 500 cal, and the 310 g of carbohydrates provides 1,240 cal, for a total of 2,460 cal/day.

A sedentary male medical student is 5′9″ tall and weighs 175 lb. Which of the following diets will allow maintenance of the current weight and also falls within current nutritional guidelines? (A) About 100 g fat, 50 g of ethanol, 100 g of protein, and 300 g of carbohydrate (B) About 80 g fat, 125 g of protein, and 310 g of carbohydrate (C) About 125 g fat, 85 g protein, and 350 g of carbohydrate (D) About 80 g fat, 60 g ethanol, 100 g protein, and 310 g carbohydrate (E) About 50 g fat, 25 g protein, and 225 g carbohydrate

*The answer is D.* A negative nitrogen balance will result from a diet deficient in one essential amino acid, or in a very diseased state. Linoleic and linolenic acids are the essential fatty acids in the diet, and a lack of these fatty acids will not affect nitrogen balance. Starch is a glucose polymer, and the lack of starch will not affect nitrogen balance. *Lysine is an essential amino acid*, whereas serine can be synthesized from a derivative of glucose. *Lack of lysine in the diet will lead to a negative nitrogen balance* as existing protein is degraded to provide lysine for new protein synthesis.

A vegan has been eating low quality vegetable protein for many years, and is now exhibiting a negative nitrogen balance. This may be occurring due to a lack of which one of the following in his/her diet? (A) Linoleic acid (B) Starch (C) Serine (D) Lysine (E) Linolenic acid

*The answer is B.* A BMI of 19.5 places the woman at the lower end of the normal range. Underweight is indicated by a BMI of ,18.5; preobesity occurs above a BMI of 25, but ,30. Class I obesity is indicated by a BMI between 30 and 35, and class II obesity by a BMI between 35 and 40.

A young woman (5' 3" tall, 1.6 m) who has a sedentary job and does not exercise consulted a physician about her weight, which was 110 lb (50 kg). A dietary history indicates that she eats approximately 100 g of carbohydrate, 20 g of protein, and 40 g of fat daily. According to the woman's BMI, into what classification does her weight and height place her? (A) Underweight (B) Normal range (C) Overweight (preobese) (D) Class I obese range (E) Class II obese range

*The answer is D.* The woman consumes 400 calories (kcal) of carbohydrate (100 g x 4 kcal/g), 80 calories of protein (20 x 4), and 360 calories of fat (40 x 9) for a total of 840 calories daily.

A young woman (5' 3" tall, 1.6 m) who has a sedentary job and does not exercise consulted a physician about her weight, which was 110 lb (50 kg). A dietary history indicates that she eats approximately 100 g of carbohydrate, 20 g of protein, and 40 g of fat daily. How many calories (kcal) does this woman consume each day? (A) 1,440 (B) 1,340 (C) 940 (D) 840 (E) 640

*The answer is C.* Because her caloric intake (840 kcal/day) is less than her expenditure (1,560 kcal/day), the woman is losing weight. She needs to increase her caloric intake. Exercise would cause her to lose more weight. She is probably in negative nitrogen balance because her protein intake is low (0.8 g/kg/day is recommended). Although her fat intake is 43% of her total calories and recommended levels are, 30%, she should increase her total calories by increasing her carbohydrate and protein intake rather than decreasing her fat intake.

A young woman (5' 3" tall, 1.6 m) who has a sedentary job and does not exercise consulted a physician about her weight, which was 110 lb (50 kg). A dietary history indicates that she eats approximately 100 g of carbohydrate, 20 g of protein, and 40 g of fat daily. On the basis of the woman's current weight, diet, and sedentary lifestyle, which one of the following does the physician correctly recommend that she should undertake? (A) Increase her exercise level (B) Decrease her protein intake (C) Increase her caloric intake (D) Decrease her fat intake to <30% of her total calories (E) Decrease her caloric intake

*The answer is B.* This woman's DEE is 1,560 calories (kcal). DEE equals BMR plus physical activity. Her weight is 110 lb/2.2 = 50 kg. Her BMR (about 24 kcal/kg) is 50 kg x 24 = 1,200 kcal/day. She is sedentary and needs only 360 additional kcal (30% of her BMR) to support her physical activity. Therefore, she needs 1,200 + 360 = 1,560 kcal each day.

A young woman (5' 3" tall, 1.6 m) who has a sedentary job and does not exercise consulted a physician about her weight, which was 110 lb (50 kg). A dietary history indicates that she eats approximately 100 g of carbohydrate, 20 g of protein, and 40 g of fat daily. What is the woman's approximate DEE in calories (kilocalories) per day at this weight? (A) 1,200 (B) 1,560 (C) 1,800 (D) 2,640 (E) 3,432

*The answer is D.* The BMI is calculated by dividing the weight of the individual (in kilograms) by the square of the height of the individual (in meters). For this woman, BMI = 50/1.6² = 19.5.

A young woman (5' 3" tall, 1.6 m) who has a sedentary job and does not exercise consulted a physician about her weight, which was 110 lb (50 kg). A dietary history indicates that she eats approximately 100 g of carbohydrate, 20 g of protein, and 40 g of fat daily. What is this woman's BMI? (A) 16.5 (B) 17.5 (C) 18.5 (D) 19.5 (E) 20.5

*The answer is D.* You are expected to know that carbohydrates have 4 Kcal/gram, proteins have 4 Kcal/gram, fat has 9 Kcal/gram, and alcohol has 7 Kcal/gram. In this question, 11 grams of fat times 9 Kcal/gram = 99 Kcal which is rounded to 100 Kcal. Dividing 100 Kcal by 4 Kcal/gram of carbohydrate is 25 grams.

After a routine physical exam and blood work, a woman with a normal weight for her height was advised that her lipid profile showed an elevation of blood triglycerides. The doctor advises the patient to lower fat consumption which disappoints her since she avidly consumes whole milk. The woman consults a nutritionist, who states that whole milk is 3.5% fat, which corresponds to approximately 11 g of fat in an 8 ounce serving. If she switch-es to drinking skim milk (nonfat), approximately how many additional grams of carbohydrates should she consume to make up for the loss of fat in the 8 ounce serving? A. 5 grams B. 11 grams C. 15 grams D. 25 grams E. 35 grams

*The answer is D.* During fasting, fatty acids are released from adipose tissue and oxidized by other cells. Liver glycogen is not depleted until about 30 hours of fasting. *After an overnight fast, both glycogenolysis and gluconeogenesis by the liver help maintain blood glucose levels.* Muscle glycogen stores are not used to maintain blood glucose levels. The liver produces ketone bodies but does not oxidize them, but under the conditions described in this question, ketone body formation would be minimal.

After a stressful week of exams, a medical student sleeps for 15 hours, then rests in bed for an hour before getting up for the day. Under these conditions, which one of the following statements concerning the student's metabolic state would be correct? (A) Liver glycogen stores are completely depleted. (B) Liver gluconeogenesis has not yet been activated. (C) Muscle glycogen stores are contributing to the maintenance of blood glucose levels. (D) Fatty acids are being released from adipose triacylglycerol stores. (E) The liver is producing and oxidizing ketone bodies to CO₂ and H₂O.

*The answer is C.* *After a high-carbohydrate meal, glucose is the major fuel for most tissues, including skeletal muscle, adipose tissue, and liver.* The increase in blood glucose levels stimulates the release of insulin, not glucagon. Insulin stimulates the transport of glucose in skeletal muscle and adipose tissue, not brain. Liver, not skeletal muscle, converts glucose to fatty acids. Although the red blood cell uses glucose as its only fuel at all times, it generates ATP from conversion of glucose to lactate, not CO₂.

After digestion of a high-carbohydrate meal, which one of the following is most likely to occur? (A) Glucagon is released from the pancreas. (B) Insulin stimulates the transport of glucose into the brain. (C) Liver and skeletal muscle use glucose as their major fuel. (D) Skeletal muscles convert glucose to fatty acids. (E) Red blood cells oxidize glucose to CO₂.

*The answer is A.* *Muscle and adipose cells require insulin to stimulate the transport of glucose into the cel*l, whereas the *glucose transporters for the blood brain barrier are always present*, and are not responsive to insulin. Thus, the brain can always utilize the glucose in circulation, whereas muscle and adipose tissue are dependent on insulin for glucose transport into the tissue.

An 18-year-old person with type 1 diabetes has not injected her insulin for 2 days. Her blood glucose is currently 600 mg/dL (normal values are 80 to 100 mg/dL). Which one of the following cells of her body can still utilize the blood glucose as an energy source? (A) Brain cells (B) Muscle cells (C) Adipose cells

*The answer is E.* Protein malnutrition, or kwashiorkor~ is characterized by an inadequate intake of protein but adequate intake of calories. Edema is the most common presenting symptom, but depigmented hair, fatty liver changes, and skin lesions are also seen. Edema is caused by decreased plasma oncotic pressure due to hypoproteinemia. This patient's history and presentation are consistent with a low-protein, normal-calorie diet. The table compares kwashiorkor and marasmus.

An 8-month-old girl is brought into the emergency department by her parents because she appears swollen. The parents weaned the child from formula 3 weeks ago. They have been giving her rice milk for 1 month. On examination, the child has a protuberant belly and 2+ pitting edema of her wrists and shins. This type of malnut rition is caused by a deficiency of what type of nutrient? A. Calorie B. Carbohydrate C. Electrolyte D. Fat E. Protein

*The answer is D.* Milk intolerance may be due to milk protein allergies during infancy, but it is commonly caused by lactase deficiency in older individuals. Intestinal lactase hydrolyzes the milk sugar lactose into galactose and glucose, both reducing sugars that can be detected as reducing substances in the stool. The symptoms of lactose intolerance (lactase deficiency) and other conditions involving intestinal malabsorption include diarrhea, cramps, and flatulence due to water retention and bacterial action in the gut. In nontropical sprue, symptoms seem to result from the production of antibodies in the blood against fragments of wheat gluten. It seems likely that a defect in intestinal epithelial cells allows tryptic peptides from the digestion of gluten to be absorbed into the blood, as well as to exert a harmful effect on intestinal epithelia. Gallbladder inflammation (cholecystitis) usually presents with acute abdominal pain (colic) with radiation to the right shoulder. The normal composition of bile is about 5% cholesterol, 15% phosphatidylcholine, and 80% bile salt in a micellar liquid form. Increased cholesterol from high-fat diets or genetic conditions can upset the delicate micellar balance, leading to supersaturated cholesterol or cholesterol precipitates that cause gallstone formation. Removal of the gallbladder is a common treatment for this painful condition. Mobilization of fats with the production of ketone bodies occurs during fasting and starvation, but ketone production is well controlled. During uncontrolled diabetes mellitus, ketogenesis proceeds at a rate that exceeds the buffering capacity of the blood to produce ketoacidosis.

An adolescent presents with abdominal discomfort, abdominal fullness, excess gas, and weight loss. Blood glucose, cholesterol, and alkaline phosphatase levels are normal. There is no jaundice or elevations. The stool tests positive for reducing substances. Which of the clinical conditions listed below is the most likely diagnosis? a. Diabetes mellitus b. Starvation c. Nontropical sprue d. Milk intolerance e. Gallstones

*The answer is C.* Severe hypoglycemia lowers the insulin level and increases glucagon. This would favor fatty acid release from the adipose and ketogenesis in the liver.

An alcoholic has been on a 2-week drinking binge during which time she has eaten little and has become severely hypoglycemic. Which additional condition may develop in response to chronic, severe hypoglycemia? A. Glycogen accumulation in the liver with cirrhosis B. Thiamine deficiency C. Ketoacidosis D. Folate deficiency E. Hyperuricemia

*The answer is A.* Following digestion, the products of digestion enter the bloodstream. These include glucose, amino acids, triacylglycerides packaged into chylomicrons from the intestine, and verylow-density lipoproteins from the liver. The hormone of anabolism, insulin, is also elevated because of the signaling of the glucose and amino acids in the blood, which allows release of insulin from the β cells of the pancreas. Insulin aids the movement of glucose and amino acids into cells. In contrast, all the hormones and energy sources associated with catabolism are decreased in the blood during this time. Long-chain fatty acids and glycerol released by lipolysis from adipocytes are not elevated. Glucagon and epinephrine are not released. The only time glucose levels rise significantly above approximately 80 mM is following a well-balanced meal when glucose is obtained from the diet. The concentration of glucose reaches a peak 30 to 45 min after a meal and returns to normal within 2 h after eating. This response of blood glucose after eating (mimicked by giving 50 g of oral glucose) is the basis for the glucose tolerance test. In the event of insulin deficiency (diabetes mellitus), the peak glucose concentration is abnormally high and its return to normal is delayed.

Approximately 3 h following a well-balanced meal, blood levels of which of the following are elevated? a. Fatty acids b. Glucagon c. Glycerol d. Epinephrine e. Chylomicrons

*The answer is C.* With alcohol at 5% wt./vol., 500 mL of beer would contain 25 g of alcohol (5 g/100 mL). Alcohol contains 7 cal/g, so 7 × 25 = 175 cal. Beer also contains some carbohydrates, so its total caloric content would be even higher than the 175 due to the ethanol alone. In contrast, 12 ounces of a cola product typically contains 150 cal (none from ethanol).

Assume that beer contains 5% wt./vol. ethanol. How many calories derived from alcohol would 500 mL (about 17 ounces) of beer contain? Choose the closest answer to your calculated value. (A) 100 (B) 140 (C) 180 (D) 220 (E) 260

*The answer is D.* The conversion of A to B is slightly unfavorable, but as soon as B is produced it will be converted to C due to the highly favorable B to C conversion (a high negative ΔG). The conversion of C to D is also highly favorable, which will lead to accumulation of D. The conversion of D to E, however, is highly unfavorable (high positive ΔG), such that D will accumulate under standard conditions.

Consider the following reaction sequence: A ↔ B ΔGo′ = +0.50 kcal/mol B ↔ C ΔGo′ = −15.50 kcal/mol C ↔ D ΔGo′ = −12.15 kcal/mol D ↔ E ΔGo′ = +21.15 kcal/mol Under standard conditions, which intermediate would accumulate? (A) A (B) B (C) C (D) D (E) E

*The answer is D.* All estimates of the BMR utilize the weight of the individual, however, adipose tissue is primarily metabolically inactive, and if an individual has a lot of adipose tissue, the contribution of the adipose tissue to the overall BMR will lead to an overestimate of the energy needs of the individual. It is not related to the amount of calories consumed, nor does it relate to use of the muscles, since the BMR is estimated for energy use during rest.

Calculation of the basal metabolic rate (BMR) for morbidly obese individuals using standard methodology is often incorrect due to which of the following? (A) Underestimation of calories consumed (B) Overestimation of calories consumed (C) Preponderance of metabolically active adipocytes (D) Preponderance of inert adipocytes (E) Reduced metabolic need of the muscles

*The answer is D.* Adipose tissue contains more calories (kilocalories) and less water than does muscle protein. Triacylglycerol stored in adipose tissue contains 9 kcal/g, and adipose tissue has about 15% water. Muscle protein contains 4 kcal/g and has about 80% water.

Consider a normal 25-year-old man, about 70 kg in weight, who has been shipwrecked on a desert island, with no food available, but plenty of freshwater. The shipwrecked man will have most of his fuel stored as triacylglycerol instead of protein in muscle due to triacylglycerol stores containing which of the following as compared to protein stores? (A) More calories and more water (B) Less calories and less water (C) Less calories and more water (D) More calories and less water (E) Equal calories and less water

*The answer is B.* In the average (70 kg) man, adipose tissue contains 15 kg of fat or 135,000 calories (kcal). Liver glycogen contains about 0.08 kg of carbohydrate (320 calories), and muscle glycogen contains about 0.15 kg of carbohydrate (600 calories). In addition, about 6 kg of muscle protein (24,000 calories) can be used as fuel. Therefore, liver glycogen contains the fewest available calories.

Consider a normal 25-year-old man, about 70 kg in weight, who has been shipwrecked on a desert island, with no food available, but plenty of freshwater. Which of the following fuel stores is least likely to provide significant calories to the man? (A) Adipose triacylglycerol (B) Liver glycogen (C) Muscle glycogen (D) Muscle protein (E) Adipose triacylglycerol and liver glycogen

*The answer is C.* Recall, ΔG = ΔGo′ + RTln ([C][D]/[A][B]). In order for the reaction to be favorable, ΔG must be negative. If one solves for the concentration of D required for ΔG = 0, then any concentration lower than the one calculated will be sufficient to allow for a negative ΔG. Thus, when ΔG is set to zero, ΔGo′ = −RT ln ([C][D]/[A][B]). Therefore, 8.65 = −(1.98 × 10−3)(298) ln ([1.25D]/[12.5]). This reduces to −14.66 = ln (D/10), or (10)(e−14.66) = [D] in mM. [D] = 4.3 × 10−6 mM, or 4.3 nM.

Consider the reaction shown below. If [A] = 5.00 mM, [B] = 2.50 mM, and [C] = 1.25 mM, what would the concentration of D have to be to allow this to be a favorable reaction under these conditions? A + B ↔ C + D ΔGo′ = +8.65 kcal/mol (A) <0.125 μM (B) <0.43 μM (C) <4.3 nM (D) <43 nM (E) <5.0 μM

*The answer is D.* Long-term exercise has been shown to increase the translocation of GLUT-4 transporters from intracellular stores to the cell surface. The GLUT-4 t ransporter is responsible for glucose uptake, thus yielding increased insulin sensitivity,

Decreased glucose uptake in muscle tissue and adipose tissue is a hallmark of insulin resistance. Moderate to extensive exercise has been shown to reduce the incidence of type 2 diabetes mellitus. What is the most likely mechanism for exercise's reduction of the incidence of diabetes? A. Decreased glucose uptake B. Increased insulin production C. Increased lipid metabolism D. Increased translocation of insulin-dependent transporters to the cell surface E. Increased translocation of insulin-independent transporters to the cell surface

*The answer is B.* Red blood cells lack mitochondria and cannot perform oxidative phosphorylation. The brain and red blood cells can only use glucose and/or ketone bodies for energy. During the fast cycle, glucose is prioritized for the brain and the red blood cells, thus glycolysis is their primary source of energy. During a long term fast (>24h), the brain relies on ketone bodies and the TCA cycle.

During a short term fast, the brain and red blood cells rely on which of the following processes for energy? (A) Oxidative phosphorylation (B) Glycolysis (C) Fatty acid oxidation (D) Gluconeogenesis

*The answer is B.* In the absorptive phase following a meal, the major source of glucose is glucose taken directly from the intestine into the blood system. Much of this glucose is absorbed into cells and, in particular, into the liver via the action of insulin, where it is stored as glycogen. Once the effects of daytime eating have subsided and all the glucose from absorption has been stored, the normal overnight fast begins. During this period, the major source of blood glucose is hepatic glycogen. Through the effects of glycogenolysis, which are mediated by glucagon, hepatic glycogen is slowly parceled out as glucose to the bloodstream, keeping blood glucose levels normal. In contrast, muscle glycogenolysis has no effect on blood glucose levels because no glucose-6-phosphatase exists in muscle and hence phosphorylated glucose cannot be released from muscle into the bloodstream. Following a more prolonged fast or in the early stages of starvation, gluconeogenesis is needed to produce glucose from glucogenic amino acids and the glycerol released by lipolysis of triacylglycerides in adipocytes. This is because the liver glycogen is depleted and the liver is forced to turn to gluconeogenesis to produce the amounts of glucose necessary to maintain blood levels.

During an overnight fast, the major source of blood glucose is a. Dietary glucose from the intestine b. Hepatic glycogenolysis c. Gluconeogenesis d. Muscle glycogenolysis e. Glycerol from lipolysis

*The answer is C.* In order to answer this question, one needs to use the Nernst equation, which equates overall changes in redox potential to Gibbs free energy. The Nernst equation is ΔGo′ = −nFΔEo′, where n is the number of electrons transferred, F is Faraday's constant, and ΔEo′ is the change in redox potential. In this case, ΔEo′ is equal to 1.02 V (the difference between −0.47 and +0.55), and n = 2 for a pair of electrons traveling through the chain. The equation thus becomes ΔGo′ = −(2)(23)(1.02) = −47 kcal/mol

For the bacterial strain referenced in the previous question, the amount of energy available from transporting a pair of electrons across this chain is which of the following? (R = 1.98 × 10−3 kcal/mol/K and F = 23 kcal/mol-V) (A) 2.3 kcal/mol (B) 23 kcal/mol (C) 47 kcal/mol (D) 70 kcal/mol (E) 100 kcal/mol

*The answer is E.* During digestion of a mixed meal, starch and other carbohydrates, proteins, and dietary triacylglycerols are broken into their monomeric units carbohydrates into simple monosaccharides, protein into amino acids, triacylglycerols into fatty acids and glycerol). *Glucose is the principal sugar in dietary carbohydrates, and thus it increases in the blood.* Amino acids and monosaccharides enter the portal vein and go to the liver first. After digestion of fats and absorption of the fatty acids, most fatty acids are converted back into triacylglycerols and subsequently into chylomicrons by intestinal cells. Chylomicrons go through lymphatic vessels and then blood, principally to adipose tissue.

During digestion of a mixed meal, which one of the following is most likely to occur? A. Starch and other polysaccharides are transported to the liver. B. Proteins are converted to dipeptides, which enter the blood. C. Dietary triacylglycerols are transported in the portal vein to the liver. D. Monosaccharides are transported to adipose tissue via the lymphatic system. E. Glucose levels increase in the blood.

*The answer is B.* Chylomicrons are the lipoprotein particles formed in intestinal epithelial cells from dietary fats, and they contain principally triacylglycerols formed from components of dietary triacylglycerols. A decreased intake of calories in general would include a decreased consumption of fat, carbohydrate, and protein, which might not lower chylomicron levels. Dietary cholesterol, although found in chylomicrons, is not their principal component.

Elevated levels of chylomicrons were measured in the blood of a patient. A dietary therapy, which decreased which one of the following answer choices would be most helpful in lowering chylomicron levels? A. Overall calories B. Fat C. Cholesterol D. Starch E. Sugar

*The answer is C.* Mr Applebod's BMR can be approximated as 24 × 111.4, or 2,673 cal/day. Since he is sedentary, his activity factor is 1.3, and has a daily caloric need of 3,475 cal/day.

Ivan Applebod is an overweight accountant, with a height of 5′9″ (1.77 m) and a weight of 245 lb (111.4 kg). As a sedentary individual, his daily caloric need approximates which of the following? (A) 2,500 cal (B) 3,000 cal (C) 3,500 cal (D) 4,000 cal (E) 4,500 cal

*The answer is A.* Recall, ΔG = ΔGο′ + RTln ([C][D]/[A][B]), so for this reaction, ΔG = 15.5 + (1.98 × 10−3)(298) ln (1.25/20). Thus, ΔG = 15.5 + (0.59) ln (0.0625). ΔG = 15.5 - 1.64 = 13.86 kcal/mol. As ΔG is a positive number, under these conditions, the reaction is still an unfavorable reaction.

Given the following reaction: A + B ↔ C + D ΔGo′ = +15.5 kcal/mol And [A] = 5 mM, [B] = 4 mM, [C] = 0.5 mM, and [D] = 2.5 mM under cellular conditions, what is the overall Gibbs free energy change for the reaction at 25°C (R =1.98 × 10−3 kcal/mol/oK) (in kcal/mol)? (A) +13.86 (B) −13.86 (C) +15.50 (D) −15.50 (E) +17.13

*The answer is C.* In the fed state, insulin will be released because of the increase in blood glucose levels. Insulin will act on muscle cells to increase glucose uptake in the muscle. Insulin will also stimulate the liver to synthesize both glycogen and fatty acids, which leads to enhanced triglyceride synthesis and very low-density lipoprotein (VLDL) production to deliver the fatty acids to other tissues of the body. Insulin will stimulate glucose uptake in fat cells, but does not stimulate fatty acid synthesis in the fat cells (i.e., unique to the liver), but will lead to enhanced triglyceride synthesis in the fat cells.

Insulin release in the fed state will lead to which of the following metabolic changes?

*The answer is A.* Centrifugation of a cellular homogenate at a force of 100,000 × g will pellet all cellular organelles and membranes. Only soluble cellular molecules found in the cytosol will remain in the supernatant. Thus, the enzymes of glycolysis and most of those of gluconeogenesis, fatty acid synthesis, and the pentose phosphate pathway will be in the supernatant. Glucose-6-phosphate dehydrogenase, which results in the formation of 6-phosphoglucono-δ-lactone from glucose-6-phosphate, is the committed step in the pentose phosphate pathway. In the pellet will be the enzymes within mitochondria, including those of the citric acid cycle (aconitase), fatty acid β oxidation (acyl CoA hydratase), and ketogenesis (hydroxybutyrate dehydrogenase). Enzymes of glycogen degradation and synthesis (glycogen synthetase) will also be in the pellet associated with glycogen particles.

If a homogenate of liver cells is centrifuged to remove all cell membranes and organelles, which of the following enzyme activities will remain in the homogenate? a. Glucose-6-phosphate dehydrogenase b. Glycogen synthetase c. Aconitase d. Acyl CoA hydratase e. Hydroxybutyrate dehydrogenase

*The answer is A.* In a prolonged state of starvation, the major source of energy for muscle is fatty acids.

In a prolonged state of starvation, which of the following is the major source of energy for muscles? A. Fatty acids B. Glucose C. Glycogen D. Ketones

*The answer is C.* The major change during prolonged fasting is that as *muscles decrease their use of ketone bodies, ketone bodies increase enormously in the blood and are used by brain as a fuel.* However, even during starvation, glucose is still required by the brain, which cannot oxidize fatty acids to an appreciable extent (thus, D is incorrect). Red blood cells can use only glucose as a fuel (thus, B is incorrect). Because the brain, red blood cells, and certain other tissues are glucose dependent, the liver continues to synthesize glucose, and blood glucose levels are maintained at only slightly less than fasting levels (thus, A is incorrect). Adipose tissue stores (approximately 135,000 kcal) are not depleted in a well-nourished individual after 1 week of fasting (thus, E is incorrect).

In a well-nourished individual, as the length of fasting increases from overnight to 1 week, which one of the following is most likely to occur? (A) Blood glucose levels decrease by approximately 50%. (B) Red blood cells switch to using ketone bodies. (C) Muscles decrease their use of ketone bodies, which increase in the blood. (D) The brain begins to use fatty acids as a major fuel. (E) Adipose tissue triacylglycerols are nearly depleted.

1. *The answer is C.* Glycogen depleted around 18 hours, gluconeogenesis from protein begins to drop gradually, and by 2 weeks, ketones have become the more important fuel for the brain. 2. *The answer is B.* Glycogen would not be mobilized from the liver. 3. *The answer is A.* Gluconeogenesis from proteins would be severely restricted without this enzyme.

In the options above, each graph depicts the primary source of fuel used by the brain during fasting/starvation. For each condition listed below, select the most closely matched graph. 1. Normal individual 2. Liver phosphorylase deficiency 3. Hepatic fructose-1,6-bisphosphatase deficiency

*The answer is C.* Free fatty acids bound to albumin are increased as a result of an increased activity of hormone-sensitive lipase in adipose tissue. Hepatic ketogenesis is stimulated by elevated levels of glucagon. The formation of acetyl CoA is increased. β-Oxidation of fatty acids in liver provides the acetyl CoA for ketogenesis.

Increased formation of ketone bodies during fasting is a result of: A. decreased levels of circulating glucagon. B. decreased formation of acetyl CoA in the liver. C. increased levels of free fatty acids in blood. D. inhibition of β-oxidation of fatty acids in the liver. E. decreased activity of hormone-sensitive lipase in adipose tissue.

*The answer is C.* To lose one pound of weight, a reduction in intake of approximately 3,500 cal is required; thus, a loss of 5 lb will require a reduction of 17,500 cal over the next 30 days (one month). Dividing 17,500 by 30 yields 583 cal/day. Since his normal intake (to maintain weight) is 3,475 cal/day, 3,475 - 583 yields 2,892 cal/day.

Ivan Applebod is an overweight accountant, with a height of 5′9″ (1.77 m) and a weight of 245 lb (111.4 kg). Which of the following diet plans will allow Mr Applebod to lose approximately 5 lb/month assuming he does not increase his activity? (A) 2,000 cal/day (B) 2,450 cal/day (C) 2,900 cal/day (D) 3,350 cal/day (E) 3,800 cal/day

1. *The answer is B.* Adipose triacylglycerols contain the largest amount of stored energy in humans, followed by protein (even though loss of too much protein will lead to death), muscle glycogen, and liver glycogen. 2. *The answer is D.* Muscle glycogen is used for energy during exercise. The glycogen is degraded to a form of glucose that can enter metabolic pathways for energy generation. Because exercise is strenuous, muscle requires large amounts of energy, and this can be generated at the fastest rate by converting muscle glycogen to pathway precursors within the muscle. Liver glycogen will produce glucose that enters the circulation. Once in the circulation, the muscle can take up that glucose and use it to generate energy; however, the rate of energy generation from liver-derived glucose is much slower than that from muscle-derived glucose. 3. *The answer is C.* Liver glycogenolysis is the major process for maintaining blood glucose levels after an overnight fast. The muscle cannot export glucose to contribute to the maintenance of blood glucose levels, and fatty acid carbons cannot be utilized for the net synthesis of glucose. 4. *The answer is A.* The nitrogen in amino acids derived from protein is converted to urea and excreted in the urine. Uric acid, another excretion product that contains nitrogen, is derived from purine bases (found in nucleic acids), not from protein.

Match each of the characteristics below with the source of stored energy that it best describes. An answer (choices A through D) may be used once, more than once, or not at all. 1. The largest amount of stored energy in the body 2. The energy source reserved for strenuous muscular activity 3. The primary source of carbon for maintaining blood glucose levels during an overnight fast 4. The major precursor of urea in the urine A. Protein B. Triacylglycerol C. Liver glycogen D. Muscle glycogen

1. *The answer is B* The brain begins to use ketone bodies when levels start to rise after 3 to 5 days of fasting. Normally, the brain will use only glucose as a fuel (most fatty acids cannot cross the blood-brain barrier to be metabolized by the brain), but when ketone bodies are elevated in the blood, they can enter the brain and be used for energy. 2. *The answer is C.* Skeletal muscle oxidizes ketone bodies, which are synthesized in the liver from fatty acids derived from adipose tissue. As the fast continues, the muscle will switch to oxidizing fatty acids, which allows ketone body levels to rise such that the brain will begin using them as an energy source. 3. *The answer is D.* Oxidation of fatty acids occurs in mitochondria. Red blood cells lack mitochondria and therefore cannot use fatty acids. The brain will not transport most fatty acids across the blood-brain barrier (the essential fatty acids are a notable exception). Therefore, the brain cannot use fatty acids as an energy source. The brain does, however, synthesize its own fatty acids, and will oxidize those fatty acids when appropriate. Red blood cells can never use fatty acids as an energy source due to their lack of mitochondria. 4. *The answer is A.* The liver converts amino acids to blood glucose by gluconeogenesis. The other substrates for gluconeogenesis are lactate from the metabolism of glucose within the red blood cells and glycerol from the breakdown of triacylglycerol to free fatty acids and glycerol. Neither the brain, nor the skeletal muscle, nor the red blood cell can export glucose into the circulation. 5. *The answer is A.* Exercising muscle produces lactate, which the liver can convert to glucose by gluconeogenesis. Blood glucose is oxidized by red blood cells and other tissues. Only the liver and kidney (to a small extent) can release free glucose into the circulation for use by other tissues.

Match each of the characteristics below with the tissue it best describes. An answer (choices A through D) may be used once, more than once, or not at all. 1. After a fast of a few days, ketone bodies become an important fuel 2. Ketone bodies are used as a fuel after an overnight fast 3. Fatty acids are not a significant fuel source at any time 4. During starvation, this tissue uses amino acids to maintain blood glucose levels 5. This tissue converts lactate from muscle to a fuel for other tissues A. Liver B. Brain C. Skeletal muscle D. Red blood cells

*The answer is D.* In general, the corresponding pathways of catabolism and anabolism are not identical (glycolysis versus gluconeogenesis, lipolysis and β oxidation of fatty acids versus fatty acid synthesis and lipogenesis, glycogenolysis versus glycogenesis). However, the citric acid cycle is a central pathway from which anabolic precursors of biosynthetic reactions may derive or into which the complete catabolism of small molecules to carbon dioxide and water may occur. For these reasons, the citric acid cycle is often called an amphibolic pathway.

Most major metabolic pathways are considered to be either mainly anabolic or catabolic. Which of the following pathways is most correctly considered to be amphibolic? a. Lipolysis b. Glycolysis c. β oxidation of fatty acids d. Citric acid cycle e. Gluconeogenesis

*The answer is C.* His protein intake of 150 kcal is about 37 g protein (150 kcal ÷ 4 kcal/g = 37 g) below the Recommended Dietary Allowance (RDA) of 0.8 g protein per kilogram body weight (thus, A is incorrect, as Otto weighs approximately 88 kg). His carbohydrate intake of 150 kcal is below the glucose requirements of his brain and red blood cells (about 150 g/day; see Chapter 2) (thus, B is incorrect). Therefore, he will be breaking down muscle protein to synthesize glucose for the brain and other glucose-dependent tissues and adipose tissue mass to supply fatty acids for muscle and tissues able to oxidize fatty acids. Because he will be breaking down muscle protein to amino acids and converting the nitrogen from both these amino acids and his dietary amino acids to urea, his nitrogen excretion will be greater than his intake and he will be in negative nitrogen balance (thus, D is incorrect). It is unlikely that he will develop hypoglycemia while he is able to supply gluconeogenic precursors.

Otto Shape, an overweight medical student (see Chapter 1), discovered that he could not exercise enough during his summer clerkship rotations to lose 2 to 3 lb/wk. He decided to lose weight by eating only 300 kcal/day of a dietary supplement that provided half the calories as carbohydrate and half as protein. In addition, he consumed a multivitamin supplement. During the first 3 days on this diet, which statement best represents the state of Otto's metabolism? A. His protein intake met the RDA for protein. B. His carbohydrate intake met the fuel needs of his brain. C. Both his adipose mass and his muscle mass decreased. D. He remained in nitrogen balance. E. He developed severe hypoglycemia.

*The answer is D.* The BMI is an estimate of the "fitness" of an individual, and is calculated by taking the weight, in kilograms, and dividing by the square of the height, in meters. Values between 18.5 and 24.9 are considered the normal range, while values above 24.9 fall into the preobese and obese categories. Body builders, whether male or female, have an increased muscle mass for their height, which adds weight. Thus, body builders will have an inflated BMI which is not indicative of their fat content (it reflects their muscle mass instead). The other individuals listed will mostly fit the criteria for a valid BMI determination.

The BMI is most likely to yield incorrect data for which of the following? (A) An anorexic 25-year-old woman (B) An obese 50-year-old man (C) A normal appearing 30-year-old man (D) A 30-year-old female bodybuilder (E) A slightly overweight 42-year-old biochemistry professor

*The answer is D.* The Recommended Daily Allowance (RDA) of a nutrient is determined from the Estimated Average Requirement (EAR) + 2 standard deviations (SD) of the mean, and should meet the needs for 97% to 98% of the healthy population. It is, therefore, a reasonable goal for the intake of a healthy individual. The EAR is the amount that prevents development of established signs of deficiency in 50% of the healthy population. Although data with laboratory animals have been used to establish deficiency symptoms, RDAs are based on data collected on nutrient ingestion by humans.

The RDA is best described by which one of the following? A. The average amount of a nutrient required each day to maintain normal function in 50% of the US population. B. The average amount of a nutrient ingested daily by 50% of the US population. C. The minimum amount of a nutrient ingested daily that prevents deficiency symptoms. D. A reasonable dietary goal for the intake of a nutrient by a healthy individual. E. It is based principally on data obtained with laboratory animals.

*The answer is C.* Ingestion of an acid or excess production by the body, such as in diabetic ketoacidosis, may induce metabolic acidosis, a condition in which both pH and HCO3− become depressed. In response to this condition, the carbonic acid-bicarbonate system is capable of disposing of the excess acid in the form of CO2. The equilibrium between bicarbonate and carbonic acid shifts toward formation of carbonic acid, which is converted to CO2 and H2O in the RBC catalyzed by carbonic anhydrase, an enzyme found mainly in the RBC. The excess CO2 is then expired by the lungs as a result of respiratory compensation for the acidosis. The main role of the kidneys in managing acidosis is through excretion of H+ rather than CO2.

The composite pKa of the bicarbonate system, 6.1, may appear to make it ill-suited for buffering blood at physiologic pH of 7.4. Nevertheless, the system is very effective at buffering against additions of noncarbonic acids. Changes in the bicarbonate/carbonic acid ratio in such cases can be regulated by: A. Recruitment of bicarbonate reserves from the peripheral tissues. B. Conversion of carbonic acid to CO2 and excretion in the urine. C. Conversion of carbonic acid to CO2 followed by removal by the lungs. D. Reaction of excess carbonic acid with the amino termini of blood proteins. E. Binding of carbonic acid by hydroxide ions from the fluid phase of blood.

*The answer is A.* High blood levels of amino acids, in addition to glucose, promote the release of insulin through their action on receptors at the surface of the β cells of the pancreas. While insulin alone could lead to a hypoglycemic effect, hypoglycemia should not be observed because glucagon is also released in response to the elevated levels of circulating amino acids. The balance of glucagon and glucose tends to keep blood levels of glucose within normal ranges while amino acid transport into cells is promoted. Due to the normal insulin levels in the fed state, ketosis and depletion of liver glycogen are not observed. Both of these events occur during fasting and starvation due to the abundance of glucagon and epinephrine in the blood as opposed to the low levels of insulin.

The jinga bean, found in the jungles of Brazil, is unique in that it is composed almost exclusively of protein. Studies have shown that, immediately following a meal composed exclusively of jinga beans, which one of the following occurs? a. A decreased release of epinephrine b. A complete absence of liver glycogen c. Hypoglycemia d. An increased release of insulin e. Ketosis caused by the metabolism of ketogenic amino acids

*The answer is B.* The order of electron flow goes from the lowest standard redox potential (E/EH2, with a value of −0.47 V) to the highest redox potential (A/AH2, with a value of +0.55 V). Redox pairs with low redox potentials are good reducing agents (they like to give up their electrons), whereas redox pairs with high redox potentials are good oxidizing agents (they love to accept electrons). Thus, the order of electron transfer would be E to D to C to B and then to A as the terminal electron acceptor.

The order of electron flow in this bacterium is which of the following? (A) A transfers to B, which transfers to C, which transfers to D, which transfers to E (B) E transfers to D, which transfers to C, which transfers to B, which transfers to A (C) C transfers to D, which transfers to E, which transfers to B, which transfers to A (D) A transfers to E, which transfers to B, which transfers to D, which transfers to C (E) E transfers to C, which transfers to A, which transfers to D, which transfers to B

*The answer is A.* The resting metabolic rate (RMR) is the calories being expended by a recently awakened resting person who has fasted for 12 to 18 hours and whose body temperature is at 20°C. *It is equivalent to the energy expenditure of our major organs and resting skeletal muscle.* Women generally have a lower RMR per kilogram body weight because more of their body weight is usually metabolically less active adipose tissue. Children have a higher RMR per kilogram body weight because more of their body weight is metabolically active organs such as brain. The RMR increases in a cold environment because more energy is being expended to generate heat. The RMR is not equivalent to our daily energy expenditure (DEE), which includes RMR, physical activity, and diet-induced thermogenesis.

The resting metabolic rate is best explained by which one of the following statements? A. It is equivalent to the caloric requirement of our major organs and resting muscle. B. It is generally higher per kilogram body weight in women than in men. C. It is generally lower per kilogram body weight in children than adults. D. It is decreased in a cold environment. E. It is approximately equivalent to the daily energy expenditure.

*The answer is A.* The ratio of conjugate base to its acid for a physiologic buffer helps determine the pH of a solution according to the terms of the Henderson-Hasselbalch equation. When the concentration of base equals that of the acid form, the ratio is 1.0 and the pH = pKa. In this case, a ratio of acid to base of 100:1 inverts to a base to acid ratio of 1:100 and calculates pH = 1.86. Such a highly acidic condition is never actually achieved within muscle cells because other weak acids, including those provided by inorganic phosphates and proteins, help buffer the solution by binding excess protons arising from dissociation of the lactic acid. pH = pKa + log₁₀( [A⁻]/[HA]) pH = 3.86 + log₁₀( [1]/[100]) pH = 3.86 + -2 pH = 1.86

The weak organic acid, lactic acid, has a pKa of 3.86. During strenuous exercise, lactic acid can accumulate in muscle cells to produce fatigue. If the ratio of the conjugate acid form lactate to the conjugate base form of lactic acid in muscle cells is approximately 100 to 1, what would be the pH in the muscle cells? A. 1.86 B. 2.86 C. 3.86 D. 4.86 E. 5.86

*The answer is C.* Insulin increases glucose transport in only two tissues, adipose and muscle. The major site of glucose uptake is muscle, which decreases hyperglycemia. Glucose and ketone transport and metabolism are insulin independent in the brain (choice D). Insulin would slow gluconeogenesis (choice A) and fatty acid release from adipose (choice B). Insulin would inhibit glycogenolysis in the liver (choice E).

Two weeks after an episode of the flu, an 8-year-old boy with IDDM is brought to the emergency room in a coma. His breathing is rapid and deep, and his breath has a fruity odor. His blood glucose is 36.5 mM (normal: 4-6 mM [70-110 mg/dL]). The physician administers IV fluids, insulin, and potassium chloride. A rapid effect of insulin in this situation is to stimulate A. gluconeogenesis in the liver B. fatty acid release from adipose C. glucose transport in muscle D. ketone utilization in the brain E. glycogenolysis in the liver

*The answer is A.* During the absorptive cycle, the insulin to glucagon ratio is high and anabolic (synthetic) processes dominate. Most tissues use glucose for energy in this state.

What is the primary sources of energy for the body's tissues during the feed cycle? (A) Glucose (B) Chylomicrons (C) Triacylglycerol (D) Amino acids (E) Branched chain amino acids (F) VLDL

*The answer is D.* If a person who has fasted overnight continues to fast for 1 week, *muscle protein will continue to decrease because it is being converted to blood glucose*. However, it will not decrease at as rapid a rate as with a shorter fast, because the brain is using ketone bodies and, therefore, less glucose. The individual's blood glucose levels will decrease abou 40%, because initially glycogenolysis and then gluconeogenesis by the liver help to maintain blood glucose levels, but oxidation of ketone bodies by the brain will reduce the brain's overall dependence on glucose. *Adipose tissue will decrease as triacylglycerol is mobilized.* Fatty acids from adipose tissue will be converted to ketone bodies in the liver. *Blood ketone body levels will rise*, and the brain will use ketone bodies as an alternative energy source, to reduce its dependency on glucose (during starvation, about 40% of the brain's energy needs can be met by oxidizing ketone bodies, whereas the other 60% still requires glucose oxidation).

When compared with an individual's state after an overnight fast, a person who fasts for 1 week will have which one of the following patterns expressed?

*The answer is E.* The change in energy states of reactants and products is indicated by E.

Which letter best represents the difference in energy between the substrates and products?

*The answer is B.* The energy required to increase the energy state of the starting material (indicated by A in the diagram) is the Gibbs free energy of Activation. The change in energy states of reactants and products is indicated by E, while D shows the maximal energy change from the energy of activation to the energy level of the products.

Which letter represents the Gibbs free energy of activation?

*The answer is A.* Malabsorption of iron, vitamin B12, or folate can be associated with anemia. Iron-deficiency anemia due to poor diet or improper uptake and/or utilization is diagnosed quite frequently. Due to blood losses during menstruation, adult women are more likely than other people to have iron-deficiency anemia. In this condition, transferrin, the iron-binding protein in plasma, is less than one-third saturated. Tetany is caused by calcium deficiency, producing muscle spasms and cramps. In the several types of porphyria, defects in heme synthesis produce excess porphyrins. The symptoms range from abdominal pain to unusual rashes and behavioral abnormalities. The hemophilias are bleeding disorders caused by clotting factor deficiencies, and hyperchylomicronemia occurs in lipoprotein lipase deficiency.

Which of the clinical conditions listed below can be associated with the malabsorption of iron? a. Anemia b. Tetany c. Hyperchylomicronemia d. Porphyria e. Hemophilia

*The answer is C.* 145 mg/dL and 120 mg/dL are elevated blood sugar levels 6 hours after a meal. The former would be diagnosed with diabetes and the latter with early, or prediabetes. 50 mg/dL after a meal is very low, which indicates that the patient is severely hypoglycemic. Normal blood glucose levels for a healthy adult under fasting conditions is between 80 mg/dL and 100 mg/dL.

Which of the following blood glucose levels is typical of a healthy adult? (A) 145 mg/dL 6 hours after a meal (B) 120 mg/dL 6 hours after a meal (C) 98 mg/dL 6 hours after a meal (D) 50 mg/dL 6 hours after a meal

*The answer is D.* The feed cycle is also called the absorptive cycle, and occurs 2-4 hours after a meal. During this period, the insulin to glucagon ratio is high, and anabolic (synthetic) processes dominate.

Which of the following describes the absorptive phase? (A) It occurs in the 2-4 hours during and after a meal (B) It occurs 2-4 hours before a meal (C) It can either be short or long term, and describes a period in which catabolic processes dominate (D) It occurs 2-4 hours after a meal, during which anabolic processes dominate

*The answer is B.* Calcium ions and calcium deposits are virtually universal in the structure and function of living things. In humans, calcium ions are required for the activity of many enzymes. Calcium is taken up from the gut in the presence of forms of vitamin D, such as cholecalciferol. Calcium is also primarily excreted through the intestine. When soluble, it is present as a divalent cation. When insoluble, it is found as hydroxyapatite (calcium phosphate) in bone. It is required by muscle cells for contraction and is sequestered into the sarcoplasmic reticulum during relaxation. It is actively transported by a calcium-ATPase across the sarcoplasmic reticulum.

Which of the following descriptions of calcium is correct? a. Calcium is abundant in the body as deposits of calcium sulfate b. Calcium ion is required as a cofactor for many reactions c. Calcium freely diffuses across the endoplasmic reticulum of muscle cells d. Calcium is most highly concentrated in muscle e. Calcium is mostly excreted by the kidney

*The answer is A.* A diet high in carbohydrate and fats spares glucose use and inhibits gluconeogenesis, thereby preventing protein catabolism and nitrogen production. A major function of the kidneys is to excrete nitrogen catabolized from proteins in the form of urea. Indeed, the major clinical measures of renal function are products of protein catabolism [blood urea nitrogen (BUN) and blood creatinine]. A diet for a patient with renal failure should therefore minimize protein and nitrogen load. Although 3 L/day of fluid is a normal intake for adults with healthy kidneys, glomerular filtration and water excretion are decreased in renal failure. Water and salt intake (particularly potassium) must therefore be limited in renal failure. Excess water or salt intake in patients with renal disease is manifest clinically by edema (swollen eyelids, swollen lower limbs).

Which of the following is appropriate for a patient with renal failure? a. High-carbohydrate diet b. High-protein diet c. Low-fat diet d. High-fiber diet e. Free water of at least 3 L per day

*The answer is A.* Type 2 diabetes is caused by a gradual resistance of peripheral tissues to insulin, while type 1 is caused by an autoimmune destruction of beta cells.

Which of the following is not a characteristic of type 2 diabetes? (A) Autoimmune destruction of beta cells (B) Gradual onset of symptoms (C) Gradual resistance of peripheral tissues to insulin (D) Most common type of diabetes

*The answer is D.* Glucagon is a peptide hormone.

Which of the following is not a key feature fo the hormone glucagon? (A) It upregulates catabolic pathways (B) It is secreted by the alpha cells in the islets of Langerhans (C) It is a counterregulatory hormone of insulin (D) It is a steroid hormone

*The answer is B.* After 3 to 5 days of starvation, the *brain begins to use ketone bodies, in addition to glucose*, as a fuel source. *Glycogen stores in the liver are depleted* (<5% of normal) during the first 30 hours of fasting. Inadequate protein in the diet results in a *negative nitrogen balance*. Blood glucose levels are being maintained by gluconeogenesis, using lactate (from red blood cells), glycerol (from triacylglycerol), and amino acids (from the degradation of muscle proteins) as carbon sources.

Which of the following metabolic patterns would be observed in a person after 1 week of starvation? Choose the one best answer.

*The answer is B and D.* Red blood cells lack mitochondria so they cannot breakdown ketone bodies for energy. The liver (and the liver alone) can produce ketone bodies from fatty acids, but it cannot use them for energy because it lacks a critical enzyme. It continues to use adipose triacylglycerol stores for energy.

Which of the following organs cannot utilize ketone bodies for energy? Select all that apply. (A) Brain (B) Red blood cells (C) Adipose (D) Liver (E) Muscle

*The answer is B.* Glucagon inhibits anabolic processes (except gluconeogenesis). Lipid synthesis (lipogenesis) is an anabolic process.

Which of the following processes would glucagon inhibit? (A) Glycogenolysis (B) Lipogenesis (C) Gluconeogenesis (D) Proteolysis

*The answer is D.* Glucagon will be secreted in response to all of the above. Its primary role is to increase blood glucose when blood glucose is low. Its secretion is activated by the fight or flight response, and it counters the effect of insulin after a protein-rich meal.

Which of the following promotes glucagon excretion? (A) Low blood glucose (B) Stress hormones (C) Elevated plasma amino acids (D) All of the above

*The answer is A.* During the feed cycle, the insulin to glucagon ratio is high and anabolic (synthetic) processes dominate. Most tissues use glucose for energy in this state.

Which of the following ratios applies to the absorptive phase? (A) A high insulin: glucagon ratio (B) A low insulin: glucagon ratio (C) An insulin: glucagon ratio of approximately 1 (D) An insulin: glucagon ratio that promotes catabolic processes

*The answer is C.* There are certain properties of metabolism that are considered truisms. (1) Futile cycles involving useless synthesis and degradation of a fuel do not occur simultaneously. (2) Acetyl CoA or substances that produce it, such as fatty acids or ketogenic amino acids, cannot be precursors of glucose. (3) ATP is a major phosphate donor and energy source; it must be present in cells at all times in order for them to function. (4) Protein phosphorylation inactivates enzymes that store glycogen and fat and activates enzymes that increase blood glucose and fatty acids. (5) Low blood glucose stimulates gluconeogenesis and glycogenolysis. (6) Low energy levels stimulate glycolysis and lipolysis. (7) High energy levels inhibit glycolysis and β oxidation of fatty acids. GTP can be a phosphate donor in reactions such as that catalyzed by phosphoenopyruvate carboxykinase during gluconeogenesis. However, ATP is more commonly used. GTP plays important roles as the energy donor for protein synthesis and in allosteric regulation/covalent modification of the G proteins involved in signal transduction.

Which of the following statements correctly describes metabolism? a. Fatty acids can be precursors of glucose b. High energy levels turn on glycolysis c. Synthesis and degradation of a substance do not occur at the same time d. Phosphorylation activates enzymes that store fat and glycogen e. Guanosine triphosphate (GTP) is the major donor for enzyme phosphorylation

*The answer is D.* Although the liver is the major site of the formation of free glucose to maintain blood glucose levels, the kidneys and intestinal epithelium (e.g., duodenum, jejunum, and ileum) may also release glucose. All of these tissues contain the enzyme glucose-6-phosphatase, an endoplasmic reticulum enzyme that dephosphorylates glucose and allows it to be transferred out of the cells. No other tissues in mammals contain this enzyme.

Which of the following tissues is capable of contributing to blood glucose? a. Skeletal muscle b. Adipose tissues c. Cardiac muscle d. Duodenal epithelium e. Cartilage

*The answer is D.* To answer this question, one needs to recall that fat contains 9 cal/g, ethanol 7 cal/g, and protein and carbohydrates 4 cal/g each. Using these numbers, diet A contains 1,875 cal, diet B contains 1,875 cal, diet C contains 1,625 cal, diet D contains 1,915 cal, and diet E contains 1,550 cal.

Which one of the following diets provides for the largest number of calories? (A) 100 g protein, 100 g fat, 100 g carbohydrate, 25 g ethanol (B) 50 g protein, 100 g fat, 150 g carbohydrate, 25 g ethanol (C) 75 g protein, 125 g fat, 50 g carbohydrate, no ethanol (D) 150 g protein, 75 g fat, 125 g carbohydrate, 20 g ethanol (E) 150 g protein, 50 g fat, 125 g carbohydrate, no ethanol

*The answer is C.* All of these patients will lose weight—the anorexic patients because of insufficient calories in the diet, the patients with type 1 DM because of low insulin levels that result in the excretion of glucose and ketone bodies in the urine, those with hyperthyroidism because of an increased BMR, and those with nontropical sprue because of decreased absorption of food from the gut. The untreated diabetic patients will have high ketone levels because of low insulin. Ketone levels may be elevated in anorexia and also in sprue, due to a reduction in levels of gluconeogenic precursors. An increased BMR would be observed only in hyperthyroidism. Nutrient malabsorption would occur only in nontropical sprue and anorexia.

Which one of the following is a common metabolic feature of patients with anorexia nervosa, untreated type 1 DM, hyperthyroidism, and nontropical sprue? (A) A high BMR (B) Elevated insulin levels in the blood (C) Loss of weight (D) Malabsorption of nutrients (E) Low levels of ketone bodies in the blood

*The answer is A.* Although the same intermediates may appear in both anabolic and catabolic pathways, one path is not simply the reverse of the other, because irreversible enzymatic steps often occur in the beginning of the reaction sequence. However, many steps in both anabolic and catabolic pathways are reversible. The same enzymes often appear in many metabolic pathways, but regulatory steps are irreversible. A number of small precursors serve as the building blocks of anabolism, while large energy-storage molecules such as glycogen, lipids, and proteins give rise to smaller molecules during catabolic processes.

Which one of the following is a correct statement about the regulation and sequence of reactions in metabolic pathways? a. The initial step in many pathways is a major determinant of control b. The sequence of steps in catabolic pathways is usually the exact reverse of the biosynthetic sequence c. Enzymes found in an anabolic pathway are rarely found in the corresponding catabolic pathway d. A small set of large precursors serves as the starting point for most biosynthetic processes in energy metabolism e. Steps in both anabolic and catabolic pathways are usually irreversible

*The answer is C.* TAG-rich chylomicrons are synthesized in (and released from) the intestine following ingestion of a meal. Glucagon is depressed in the absorptive period. Acetoacetate, free fatty acids, and lactate are not elevated.

Which one of the following is elevated in plasma during the absorptive (fed) period as compared with the postabsorptive (fasted) state? A. Glucagon. B. Acetoacetate. C. Chylomicrons. D. Free fatty acids. E. Lactate.

*The answer is B.* In humans, arginine is classified as a semiessential or conditionally essential amino acid, depending on the developmental stage and health status of the individual. Preterm infants are unable to synthesize or create arginine internally, making the amino acid nutritionally essential for them.

Which one of the following is semiessential amino acid for humans? (A) Valine (B) Arginine (C) Lysine (D) Tyrosine

*The answer is D.* The carbon skeletons of glucogenic amino acids are used by the liver for gluconeogenesis. Liver glycogen is nearly depleted by 12 hours after a meal, and muscle glycogen cannot give rise to free glucose because muscle lacks glucose 6-phosphatase. Acetoacetate is metabolized to acetyl CoA, which is not glucogenic. Lactate can arise from anaerobic glycolysis in muscle and red blood cells, but is less important than amino acids as a source of glucose.

Which one of the following is the most important source of blood glucose during the last hours of a 48-hour fast? A. Muscle glycogen. B. Acetoacetate. C. Liver glycogen. D. Amino acids. E. Lactate.

*The answer is A.* *The brain requires glucose because fatty acids cannot readily cross the blood-brain barrier to enter neuronal cells.* Thus, glucose production is maintained at an adequate level to allow the brain to continue to burn glucose for its energy needs. The other organs listed as possible answers can switch to the use of alternative fuel sources (lactate, fatty acids, amino acids) and are not as dependent on glucose for their energy requirements as is the brain.

Which one of the following organs has the highest demand for glucose as a fuel? (A) Brain (B) Muscle (skeletal) (C) Heart (D) Liver (E) Pancreas

*The answer is B.* The increased insulin and decreased glucagon levels characteristic of the fed state promote the synthesis of fructose 2,6- bisphosphate. Most covalently modified enzymes are in the dephosphorylated state and are active. Acetyl CoA is not elevated in the fed state. The transport of glucose in the liver is not insulin sensitive. Synthesis of glucokinase is enhanced in the fed state.

Which one of the following statements concerning the fed state is correct? A. Most enzymes that are regulated by covalent modification are in the phosphorylated state. B. Hepatic fructose 2,6-bisphosphate is elevated. C. The oxidation of acetyl CoA is increased. D. Insulin stimulates the transport of glucose into hepatocytes. E. The synthesis of glucokinase is repressed.

*The answer is D.* Glucose is the major fuel for the brain in the well-fed state. The brain requires a continuous supply of glucose at all times. In fact, if glucose drops to a low level, convulsions may follow. However, during starvation or fasting, the brain is capable of obtaining approximately 75% of its energy from circulating ketone bodies. During the absorptive phase, ketone bodies such as acetoacetate and 3-hydroxybutyrate are low. Circulating amino acids are utilized for protein synthesis. Liver production of NADPH is at a high level because it is needed for fatty acid synthesis. Glucose is actively transported into all cells, including adipocytes, which require it to form glucose-3-phosphate for esterifying fatty acids into triacylglyceride.

Which one of the following statements is correct regarding the well-fed state? a. NADPH production by the hexose monophosphate shunt is decreased b. Acetoacetate is the major fuel for muscle c. Glucose transport into adipose tissue is decreased d. The major fuel used by the brain is glucose e. Amino acids are utilized for glucose production

*The answer is F.* Acetyl-CoA, a product of glucose breakdown, is a substrate for fatty acid synthesis. Thus, excess dietary carbohydrates can be stored as glycogen in the liver and muscle and as body fat in the liver and adipose.

Which organs can store carbohydrates as fats? (A) Adipose tissue (B) Skeletal muscle (C) Liver (D) Brain (E) B and C (F) A and C

*The answer is E.* In a normal postabsorptive patient, blood fuel values are 4.5 mM glucose, 0.5 mM free fatty acids, 0.02 mM ketone bodies, and 4.5 mM amino acids (choice d). Levels of ketone bodies are always low in a fed person. Following several days of starvation, a catabolic homeostasis has set in, such that free fatty acids (1.5 mM) have risen and production of ketone bodies (5 mM) by the liver is proceeding (choice b). At this point, glycogen stores have been depleted. Much of the blood glucose, which is maintained at about 4.5 mM throughout starvation, now comes from gluconeogenesis using increased concentrations of amino acids (4.7 mM) derived from protein breakdown. Most of the brain's fuel supply still derives from glucose at this time. Since the brain accounts for at least 20% of the body's total consumption of fuel, this amount can be considerable. Following prolonged starvation, utilization of glucose and hence catabolism of protein are spared by the induction of increased amounts of brain enzymes to utilize ketone bodies. Thus, in prolonged starvation, the blood concentration of amino acids (3.1 mM) decreases, whereas that of free fatty acids (2 mM) and ketone bodies (8 mM) increases (choice e). Of course, blood glucose is maintained at about 4.5 mM. The lack of insulin in diabetics causes a stimulation of lipolysis, glycogenolysis, gluconeogenesis, and ketogenesis. Thus, the blood values of free fatty acids (2 mM), ketone bodies (10 mM), and amino acids (4.5 mM) should resemble those of a fasting or starving person with one major exception—the high level of blood glucose [12 mM (choice c)]. The lack of insulin does not allow the glucose to enter most cells.

Which set of blood values most closely correlates with a patient who has conducted a hunger strike for 1 month?

*The answer is B.* Gluconeogenic substrates include amino acids from skeletal muscle, so ketone bodies help prevent the degradation of essential proteins by reducing the need for more glucose.

Why does the liver produce ketone bodies during fasting when it can just produce glucose via gluconeogenesis? (A) Ketone bodies spare glucose for the peripheral tissues (B) Gluconeogenic substrates include amino acids from skeletal muscle; ketogenesis prevents the degradation of essential proteins (C) Ketone bodies promote the degradation of muscle proteins, and therefore spare nutrients for the brain and red blood cells. (D) None of the above.

*The answer is B.* The BMI is calculated as the weight of the person (in kg) divided by the height squared (in meters). Thus, for this patient, the BMI is equal to 45.85 divided by (1.67)2, which is 16.44. The BMI stands for body mass index, and can be used to estimate body fat content. A value of <18.5 is considered underweight, values between 18.5 and 24.9 are considered in the normal range, values of 25 through 29.9 are considered overweight, and values of 30 or greater are considered obese. Values of 40 or more are considered morbidly obese, whereas values between 35 and 40 are considered clinically obese. The formulas in order to perform these calculations are summarized in a figure above. The second figure allows one to utilize a graph to calculate the BMI.

You see in your office a thin, anxious woman who is concerned about her weight. She is worried that she may have a parasite causing her to lose weight. She stands 5′5′′ tall (1.67 m) and weighs 101 lb (45.85 kg). The same patient then describes to you a typical day of eating, which consists of 250 g of carbohydrates, 10 g of fat, and 100 g of protein. She denies any ethanol intake. She also exercises about 2h/day. An estimate of her body mass index (BMI) is which of the following? (A) 14 (B) 16 (C) 18 (D) 20 (E) 22

*The answer is C.* The BMR can be estimated by multiplying the weight (in kg) times 24 cal/kg, which is assuming an energy use of 1 cal/h/kg. Multiplying 45.85 times 24 yields 1,100 cal/day. Her metabolic need can be estimated by multiplying her BMR times an acitivty factor (how active the individual is). For someone who exercises 2 h/day (moderately active) the activity factor is 1.6, so her daily caloric needs are 1,100 × 1.6, or 1,760 cal/day. More accurate representations of the BMR can be obtained from using the formulas in Table 8-2, although for the purposes of this textbook the approximation of 24 cal/kg/day will be utilized for both males and females of all ages.

You see in your office a thin, anxious woman who is concerned about her weight. She is worried that she may have a parasite causing her to lose weight. She stands 5′5′′ tall (1.67 m) and weighs 101 lb (45.85 kg). The same patient then describes to you a typical day of eating, which consists of 250 g of carbohydrates, 10 g of fat, and 100 g of protein. She denies any ethanol intake. She also exercises about 2h/day. For the same patient, her daily caloric needs can be estimated to be which of the following? (A) 1,250 (B) 1,500 (C) 1,750 (D) 2,000 (E) 2,250

*The answer is D.* The BMI of 16.4 places the individual in an underweight situation, and she is currently consuming fewer calories (1,490) per day than she requires (1,760), which will lead to weight loss, and further exacerbate her underweight condition. You have a diagnosis, there is no reason to pursue further testing. She can be counseled on how to gain weight.

You see in your office a thin, anxious woman who is concerned about her weight. She is worried that she may have a parasite causing her to lose weight. She stands 5′5′′ tall (1.67 m) and weighs 101 lb (45.85 kg). The same patient then describes to you a typical day of eating, which consists of 250 g of carbohydrates, 10 g of fat, and 100 g of protein. She denies any ethanol intake. She also exercises about 2h/day. Given this same patient's eating habits and lifestyle, which of the following best describes her metabolic state? (A) She is gaining weight (B) She is in caloric balance (C) She is in the healthy range of BMI but is losing weight (D) She is in an unhealthy range of BMI and is losing weight (E) She has a tapeworm and needs lab testing

*The answer is B.* Carbohydrates contain 4 cal/g, protein also contains 4 cal/g, and fat contains 9 cal/g (because it is more reduced than either protein or carbohydrates). Given the patient's diet, she is consuming (250 × 4) + (10 × 9) + (100 × 4), or 1,490 cal/day.

You see in your office a thin, anxious woman who is concerned about her weight. She is worried that she may have a parasite causing her to lose weight. She stands 5′5′′ tall (1.67 m) and weighs 101 lb (45.85 kg). The same patient then describes to you a typical day of eating, which consists of 250 g of carbohydrates, 10 g of fat, and 100 g of protein. She denies any ethanol intake. She also exercises about 2h/day. Her daily caloric intake is about which of the following? (A) 1,250 (B) 1,500 (C) 1,750 (D) 2,000 (E) 2,250


संबंधित स्टडी सेट्स

Chapter 9 (The Market Revolution, 1800 - 1840)

View Set

تاريخ الأردن الوحدة الثانية، ف ١

View Set

Problem Based Learning II Basic Science Test 2

View Set

Marketing Midterm 3401 LSU, MKT 3401 Exam 1 (LSU-Karam)

View Set

MKT320 Chapter 12 Guidepost Questions

View Set